Difference between revisions of "2024 AMC 12B Problems/Problem 20"

(Problem 20)
(Solution 1)
Line 11: Line 11:
  
 
==Solution #1 ==
 
==Solution #1 ==
Let midpoint of BC as M, extends AM to D and MD = x,  
+
Let midpoint of <math>BC</math> as <math>M</math>, extends <math>AM</math> to <math>D</math> and <math>MD=x</math>,  
  
triangle ACD  has 3 sides (40,42,2x)
+
triangle <math>ACD</math> has <math>3</math> sides <math>(40,42,2x)</math>
as such, 2<  2x < 82  
+
as such,<cmath>2<  2x < 82</cmath>
1<= x <=41
+
<cmath>1\le x \le41</cmath>
so p = 1, q=41  
+
so <cmath>p = 1, q=41</cmath>
  
2*Area(ABC) =  40 * 42 * sin(A) <= 2*840
+
<cmath>2\cdot f(x) =  40 \cdot 42 \cdot \sin(A) \le 2\cdot840</cmath>
so r =  max(Area{ABC)) = 840  
+
so <math>r = 840 </math>
which is achieved when A = 90 degree , then angle ACD = 90 degree,
+
which is achieved when <math>A = 90^\circ</math> , then <math>\angle ACD = 90^\circ</math>
<math>(2x)^2 = 40^2 + 42^2 </math>
+
<cmath>(2x)^2 = 40^2 + 42^2 </cmath>
x = 29  
+
<cmath>x = 29</cmath>
s= 29  
+
<cmath>s= 29 </cmath>
p+q+s+r = 1 + 41 + 29 + 840 = <math>\fbox{\textbf{(C) } 911}</math>
+
<cmath>p+q+s+r = 1 + 41 + 29 + 840 = \fbox{\textbf{(C) } 911}</cmath>
  
 
~[https://artofproblemsolving.com/wiki/index.php/User:Cyantist luckuso]
 
~[https://artofproblemsolving.com/wiki/index.php/User:Cyantist luckuso]
 
  
 
==See also==
 
==See also==
 
{{AMC12 box|year=2024|ab=B|num-b=19|num-a=21}}
 
{{AMC12 box|year=2024|ab=B|num-b=19|num-a=21}}
 
{{MAA Notice}}
 
{{MAA Notice}}

Revision as of 03:15, 14 November 2024

Problem 20

Suppose $A$, $B$, and $C$ are points in the plane with $AB=40$ and $AC=42$, and let $x$ be the length of the line segment from $A$ to the midpoint of $\overline{BC}$. Define a function $f$ by letting $f(x)$ be the area of $\triangle ABC$. Then the domain of $f$ is an open interval $(p,q)$, and the maximum value $r$ of $f(x)$ occurs at $x=s$. What is $p+q+r+s$?

$\textbf{(A) }909\qquad \textbf{(B) }910\qquad \textbf{(C) }911\qquad \textbf{(D) }912\qquad \textbf{(E) }913\qquad$

Solution #1

Let midpoint of $BC$ as $M$, extends $AM$ to $D$ and $MD=x$,

triangle $ACD$ has $3$ sides $(40,42,2x)$ as such,\[2<  2x < 82\] \[1\le x \le41\] so \[p = 1, q=41\]

\[2\cdot f(x) =   40 \cdot 42 \cdot \sin(A) \le 2\cdot840\] so $r = 840$ which is achieved when $A = 90^\circ$ , then $\angle ACD = 90^\circ$ \[(2x)^2 = 40^2 + 42^2\] \[x = 29\] \[s= 29\] \[p+q+s+r = 1 + 41 + 29 + 840 = \fbox{\textbf{(C) } 911}\]

~luckuso

See also

2024 AMC 12B (ProblemsAnswer KeyResources)
Preceded by
Problem 19
Followed by
Problem 21
1 2 3 4 5 6 7 8 9 10 11 12 13 14 15 16 17 18 19 20 21 22 23 24 25
All AMC 12 Problems and Solutions

The problems on this page are copyrighted by the Mathematical Association of America's American Mathematics Competitions. AMC logo.png